Study guides, Class notes & Summaries

Looking for the best study guides, study notes and summaries about ? On this page you'll find 218 study documents about .

Page 2 out of 218 results

Sort by

LSAT practice questions and answer rated A+ 2024
  • LSAT practice questions and answer rated A+ 2024

  • Exam (elaborations) • 1 pages • 2024
  • Available in package deal
  • LSAT practice questions and answer rated A+ 2024 Although the charter of Westside state that the student body must include some students with special education needs, no students with learning disabilities have yet enrolled in the school. Therefore, the school is currently in violation of its charter. The conclusion of the argument follows logically if which of the following is assumed? A) All students with learning disabilities have special ed need B) The school currently has no student ...
    (0)
  • $14.49
  • + learn more
LSAT PRACTICE QUESTIONS with complete solution 2024
  • LSAT PRACTICE QUESTIONS with complete solution 2024

  • Exam (elaborations) • 5 pages • 2024
  • Available in package deal
  • LSAT PRACTICE QUESTIONS with complete solution 2024 LOGICAL REASONING: IDENTIFY THE CENTRAL ASSUMPTION: Linsey has been judged to be a bad songwriter simply because her lyrics typically are disjointed and subjective. This judgment is ill founded, however, since the writings of many modern novelists typically are disjointed and subjective and yet these novelists are widely held to be good writers. Which one of the following is an ASSUMPTION on which the argument depends? (A) Disjoint...
    (0)
  • $14.59
  • + learn more
LSAT prep Question and answer rated A+ 2024
  • LSAT prep Question and answer rated A+ 2024

  • Exam (elaborations) • 7 pages • 2024
  • Available in package deal
  • LSAT prep Question and answer rated A+ 2024LR assumption strategy - correct answer Bridge the gap between the evidence and the conclusion. LR assumption Q stem idetifiers - correct answer "assumed," "assumption," "presuppose," "if added to the premises." LR strengthen/weaken strategy - correct answer Find the answer that makes the conclusion more or less likely to follow from the evidence. LR strengthen/weaken Q stem identifiers - correct answer "strengthen," "mos...
    (0)
  • $13.99
  • + learn more
LSAT Prep Question with complete solution 2024
  • LSAT Prep Question with complete solution 2024

  • Exam (elaborations) • 16 pages • 2024
  • Available in package deal
  • LSAT Prep Question with complete solution 2024Lesson 1 - correct answer ... Argument - correct answer Consists of a conclusion and a set of premises given in support of that conclusion Set of Facts - correct answer A group of statements from which no conclusion is drawn. Valid Conclusion - correct answer A statement that must be true according to the premises Invalid Conclusion - correct answer A statement that is not necessarily true according to the premises Assum...
    (0)
  • $13.99
  • + learn more
LSAT Prep Questions and answers 2024 verified to pass
  • LSAT Prep Questions and answers 2024 verified to pass

  • Exam (elaborations) • 21 pages • 2024
  • Available in package deal
  • LSAT Prep Questions and answers 2024 verified to passLOGICAL: Laird: Pure research provides us with new technologies that contribute to saving lives. Even more worthwhile than this, however, is its role in expanding our knowledge and providing new, unexplored ideas. Kim: Your priorities are mistaken. Saving lives is what counts most of all. Without pure research, medicine would not be as advanced as it is. A) Laird and Kim disagree on whether pure research derives its significance in p...
    (0)
  • $13.99
  • + learn more
LSAT Prep Test Question and answers already passed 2024
  • LSAT Prep Test Question and answers already passed 2024

  • Exam (elaborations) • 6 pages • 2024
  • Available in package deal
  • LSAT Prep Test Question and answers already passed 2024 S1Q1: French divers recently found a large cave along the coasr of the Mediterrian Sea. The cave is accessible only through an underwater tunnel. The interior of the cave is completely filled with seawater and contains numerous stalagmites, which are stony pillars that form when drops of water fall repeadetely on a single spot on a cave floor, leaving behind mineral deposits that accumulate over time. - correct answer The information p...
    (0)
  • $14.99
  • + learn more
LSAT Question and answer 100% correct 2024
  • LSAT Question and answer 100% correct 2024

  • Exam (elaborations) • 4 pages • 2024
  • Available in package deal
  • LSAT Question and answer 100% correct 2024 Split block - correct answer When thre rule can occur both way either S first or T first (denoted by circle and two options) Rotating split blocks - correct answer When can occur either way but spaces between two letters (Denoted r/s_ _r/s) in square) Not blocks - correct answer This can't go next to here (Denoted R big / over whole box s) Must be true - correct answer same thing as cannot be false logical opposite of no...
    (0)
  • $14.49
  • + learn more
LSAT Question Stems with complete solution 2024
  • LSAT Question Stems with complete solution 2024

  • Exam (elaborations) • 5 pages • 2024
  • Available in package deal
  • LSAT Question Stems with complete solution 2024 Assumption - correct answer Which of the following is an assumption on which the argument depends? Assumption - correct answer Which of the following is an assumption on which the argument relies? Assumption - correct answer The argument above relies on which of the following assumptions? Assumption - correct answer The argument above depends on which of the following assumptions? Assumption - correct answer In order for ...
    (0)
  • $13.99
  • + learn more
LSAT Question Types with complete solutions 2024
  • LSAT Question Types with complete solutions 2024

  • Exam (elaborations) • 3 pages • 2024
  • Available in package deal
  • LSAT Question Types with complete solutions 2024 Type 1: Must Be True/ Conclusion/ Inference - Type 1v= "Most Strongly Supported" 13.7% of LR - correct answer If statements above are true, then which of the following must also be true? Which one of the following is most strongly supported by the information above? Type 2: Strengthen/ Premise/ Weaken - Type 2 - Type 2n - Type 2s 27.2% of LR - correct answer The argument above would be most strengthened if which one o...
    (0)
  • $13.49
  • + learn more
LSAT VOCAB Question and answer 100% correct 2024
  • LSAT VOCAB Question and answer 100% correct 2024

  • Exam (elaborations) • 18 pages • 2024
  • Available in package deal
  • LSAT VOCAB Question and answer 100% correct 2024 apathy - correct answer lack of emotion anecdote - correct answer interesting, amusing story acquiesce - correct answer to give in, agree acuity - correct answer ability to understand ascetic - correct answer self-denying (w/o indulgence or luxury) apprehension - correct answer anxiety, fear advocated - correct answer to speak or write in favor autonomy - correct answer self-governing, independence a...
    (0)
  • $13.49
  • + learn more